Fix typos in Serway and Jewett v8's 23.62 and 25.21.
[course.git] / latex / problems / Serway_and_Jewett_8 / problem25.21.tex
index 403a08fc34ef37036265b89e039ec93e7778e871..116769bb2850cc55467c4a06644a3e3645333153 100644 (file)
@@ -1,10 +1,10 @@
 \begin{problem*}{25.21}
-Two particles each with charge $+2.00\U{$\mu$C}$ are lopcated on the
+Two particles each with charge $+2.00\U{$\mu$C}$ are located on the
 $x$ axis.  One is at $x=1.00\U{m}$, and the other is at
 $x=-1.00\U{m}$.  \Part{a} Determine the electric potential on the $y$
 axis at $y=0.500\U{m}$.  \Part{b} Calculate the change in electric
 potential energy of the system as a third charged particle of
-$-3.00\U{$\mu$C}$ is brought in from infinitely far away to a positino
+$-3.00\U{$\mu$C}$ is brought in from infinitely far away to a position
 on the $y$ axis at $y=0.500\U{m}$.
 \end{problem*}